Series Boundedness: A Challenging Mathematical Question

In summary, the conversation discusses a series and its boundedness. The series is divergent, but the sum of any two consecutive terms goes to zero as O(1/n^2). The speaker highlights the use of Taylor expansion to prove the boundedness of the series and provides an example of using it to approximate the series. The speaker also mentions that the problem is difficult to solve.
  • #1
emptyboat
28
1
I asked by someone.
But I can't answer to it.
[tex]\sum^{\infty}_{1}(-1)^n*(1+\frac{1}{n})^n[/tex]
Is this series bounded?
I can't do anything about that.
 
Last edited:
Physics news on Phys.org
  • #2
Do you know what

[tex]\lim_{n \rightarrow \infty} \left(1 + \frac{1}{n}\right)^n[/tex]

equals?
 
  • #3
I know this series diverge by lim an=e, not zero.
But I can't figure out it's bounded.
Would you give me more informations?
 
Last edited:
  • #4
The sum of any two consecutive terms goes to zero as O(1/n^2). Since the sum of 1/n^2 is convergent, you can prove that your original series is bounded.
 
  • #5
hmm... Do you mean (1 + 1/(n+1))^(n+1) - (1 + 1/n)^n ≤1/n^2?
I can not understand it. Give me more information, please.
 
Last edited:
  • #6
emptyboat said:
hmm... Do you mean (1 + 1/(n+1))^(n+1) - (1 + 1/n)^n ≤1/n^2?
I can not understand it. Give me more information, please.

Correct. You need to prove that there's such N and C that, for all n>N,

(1 + 1/(n+1))^(n+1) - (1 + 1/n)^n ≤ C/n^2

You can do that if you write

(1+1/n)^n = exp(n log (1+1/n))

and then use Taylor expansion of log.
 
Last edited:
  • #7
I computed the process you have given.
log(x)=(x-1) - 1/2*(x-1)^2 + 1/3*(x-1)^3 - ...(correct?)
So My result is
exp (1 - 1/2(n+1) + 1/3(n+1)^2 - ...)
- exp (1 - 1/2n + 1/3n^2 - ...)
let exp(A(n+1)) - exp((An))
exp does not linear, I can't do further.
Help me more...
 
  • #8
[tex]exp(1 - \frac{1}{2n} + \frac{1}{3n^2} + ...) \approx e*( - \frac{1}{2n} + \frac{1}{3n^2}) + ( - \frac{1}{2n} + \frac{1}{3n^2})^2/2 + ...) = e*(-\frac{1}{2n}+\frac{1}{3n^2} + \frac{1}{8n^2} + ...) = e*(-\frac{1}{2n}+\frac{11}{24n^2} + ... )[/tex]

where ... are terms that go down as 1/n^3 or faster.

[tex]|\frac{1}{2(n+1)} - \frac{1}{2n}| = \frac{1}{2n(n+1)} < \frac{1}{2n^2}[/tex]
 
  • #9
Oh I see!
Thank you for your advice.
I think this problem is very hard to solve!
 

What does it mean for a series to be bounded?

A series is considered bounded if its terms do not grow infinitely larger or smaller as the series progresses. In other words, there is a limit to how much the terms can increase or decrease.

How can I determine if a series is bounded?

To determine if a series is bounded, you can look at the behavior of the terms as the series progresses. If the terms approach a finite limit or fluctuate within a certain range, the series is considered bounded. Alternatively, you can also use mathematical tests such as the ratio test or the comparison test to determine if a series is bounded.

What are the consequences of a series being bounded?

If a series is bounded, it means that the series will converge to a finite value. This makes it easier to analyze and work with mathematically. Additionally, a bounded series is more likely to have practical applications and real-world significance.

Can a series be both bounded and divergent?

Yes, it is possible for a series to be bounded but still diverge, meaning that the terms do not approach a finite limit as the series progresses. This can happen if the terms fluctuate within a certain range, but the range itself does not approach a finite limit.

What are some examples of bounded and divergent series?

An example of a bounded series would be the alternating harmonic series, where the terms oscillate between positive and negative values but ultimately approach a finite limit. An example of a divergent series would be the harmonic series, where the terms continue to grow larger and larger without approaching a finite limit.

Similar threads

Replies
3
Views
241
Replies
2
Views
1K
Replies
6
Views
682
Replies
15
Views
2K
Replies
14
Views
2K
Replies
2
Views
789
Replies
7
Views
1K
Replies
3
Views
946
Replies
3
Views
980
  • Calculus
Replies
3
Views
2K
Back
Top